আংশিক উচিত


12

নিম্নলিখিতটি mtcarsডেটাসেট থেকে তৈরি একটি মডেল :

> ols(mpg~wt+am+qsec, mtcars)

Linear Regression Model

ols(formula = mpg ~ wt + am + qsec, data = mtcars)

                Model Likelihood     Discrimination    
                   Ratio Test           Indexes        
Obs       32    LR chi2     60.64    R2       0.850    
sigma 2.4588    d.f.            3    R2 adj   0.834    
d.f.      28    Pr(> chi2) 0.0000    g        6.456    

Residuals

    Min      1Q  Median      3Q     Max 
-3.4811 -1.5555 -0.7257  1.4110  4.6610 

          Coef    S.E.   t     Pr(>|t|)
Intercept  9.6178 6.9596  1.38 0.1779  
wt        -3.9165 0.7112 -5.51 <0.0001 
am         2.9358 1.4109  2.08 0.0467  
qsec       1.2259 0.2887  4.25 0.0002  

মডেলটি 0.85 এর মোট দিয়ে ভাল বলে মনে হচ্ছে । যাইহোক, নিম্নলিখিত প্লটের উপর আংশিক আর 2 টি মান দেখা যায় তবে এই মানটি জুড়ে না। তারা প্রায় 0.28 পর্যন্ত যোগ করে।R2R2

> plot(anova(mod), what='partial R2')

এখানে চিত্র বর্ণনা লিখুন

সমস্ত আংশিক এবং মোট আর 2 এর যোগফলের মধ্যে কোনও সম্পর্ক আছে ? বিশ্লেষণ প্যাকেজ দিয়ে করা হয় ।R2R2rms


1
(অ্যামিবার খুব ভাল উত্তর ছাড়াও) আংশিক সম্পর্ক সম্পর্কিত stats.stackexchange.com/q/76815/3277 বনাম মানক রেগ্রেশন সহগ সম্পর্কে একটি ঘনিষ্ঠ প্রশ্ন ।
ttnphns

উত্তর:


11

না।

R2R2

R2R2

R2=1R21

R2R2R2

R2


একটি খুব স্পষ্ট ব্যাখ্যা জন্য ধন্যবাদ। এই প্রশ্নের পরিস্থিতিতেও এটি ঘটতে পারে: stats.stackexchange.com/questions/155447/… । তারপরে আংশিক আর ^ 2 কি পৃথক ভবিষ্যদ্বাণীকের গুরুত্ব বা অবদানের যুক্তিসঙ্গত সূচক? অথবা আপনি 'অনুপাত আর ^ 2' বা 'অবশিষ্ট আর ^ 2' বা 'চিসক' বা 'চিস্ক বিয়োগে ডিএফ' বা 'অনুপাতের চিস্ক' বা 'আইসি'র মতো আরও কিছু প্রস্তাব দিতে চান? এগুলি সমস্ত আরএমএস প্যাকেজে উপলব্ধ। না মানক সহগ?
rnso

হ্যাঁ, আমি নিশ্চিত নই যে কেন এই প্রশ্নটি অস্পষ্ট হিসাবে ধরে রাখা হয়েছিল; আমি মনে করি এটি পরিষ্কার (এবং প্রায় এটির একটি সদৃশ তবে তর্কযোগ্যভাবে যথেষ্ট নয়)। ভবিষ্যদ্বাণীকারীদের গুরুত্বের যুক্তিসঙ্গত সূচকগুলি সম্পর্কে: আমি আমার উত্তরে আমি যুক্ত থ্রেডটি পড়ার জন্য আপনাকে দৃ strongly়ভাবে আমন্ত্রণ জানাচ্ছি, যা এই প্রশ্নের সম্পর্কে অবিকল রয়েছে । সেখানে আমার নিজস্ব উত্তরও রয়েছে, যেখানে আমি বেশ কয়েকটি বিভিন্ন সূচকের সংক্ষিপ্ত বিবরণ করি। তাদের সকলের বিভিন্ন ত্রুটি রয়েছে; দেখে মনে হচ্ছে এই সমস্যার কোনও নিখুঁত সমাধান নেই (এবং এটি হতে পারে না)।
অ্যামিবা

আমি এটা স্পষ্ট যেমন হোল্ড উপর করা কারণ ব্যাখ্যা আদৌ কিভাবে দুটি মডেল বিরোধ দেওয়া হয়। সম্ভবত আমি সেই নির্দিষ্ট পরিস্থিতির জন্য একটি উত্তর চেয়েছিলাম ভেবে ভুল ছিলাম। এই উত্তরটি দেওয়া , আপনারা কি মনে করেন যে এটির পুনরায় খোলার প্রয়োজন?
স্কর্চচি - মনিকা পুনরায় ইনস্টল করুন

সুতরাং আংশিক আর 2 একটি গ্রাফের মধ্যে তুলনীয় এবং 2 গ্রাফের মধ্যে নয়। এছাড়াও 'আংশিক আর 2' মোট মোট আর 2 এর অবদানকে ইঙ্গিত দেয় না এবং তাই আমি মনে করি এটি একটি ভুল নাম। আমি এখানে উত্তর পেয়েছি বলে এখনই অন্য প্রশ্নের দরকার নেই।
rnso

আমি সমস্ত উত্তর upvoting হয়েছে। তারপরেও আমি আমার বেশিরভাগ প্রশ্ন গ্রহণ করেছি।
rnso
আমাদের সাইট ব্যবহার করে, আপনি স্বীকার করেছেন যে আপনি আমাদের কুকি নীতি এবং গোপনীয়তা নীতিটি পড়েছেন এবং বুঝতে পেরেছেন ।
Licensed under cc by-sa 3.0 with attribution required.